Complex Numbers - Gaussian Integers, Gaussian Primes

Intro to Complex Numbers in problems, and applied Gaussian Integers.

Background/Definitions

  • The set of complex numbers \(\mathbb{C}[i]\) consists of all expressions \(\text{a} + \text{b}i\) where \(\text{a, b}\) are real numbers and \(i\) is the imaginary unit.
  • Every complex number \(z = \text{a} + \text{b}i\) can be mapped to point \((\text{a}, \text{b})\) in the complex plane. Likewise, \(z\) can be expressed in polar form \(re^{i\theta} = r(\cos\theta + i \sin\theta)\) or \(r\text{cis }\theta\), where \((r, \theta)\) is the polar form of \((\text{a}, \text{b})\).
  • \(r = \sqrt{a^2+b^2}\) is called the modulus or norm of \(z\), \(\lvert z \rvert\). \(\theta\), the angle that segment \(\text{OZ}\) makes with the terminal position, is called the argument, \(\arg(z)\).
  • The conjugate of \(z\), \(\overline{z}\), is the reflection of \(z\) over the real axis, or \(\text{a} - \text{b}i\).

Exercises:

  • Find \(r\) and \(\theta\) for \(z = \frac{-1+\sqrt{3}i}{4}\).
  • Find all \(z\) such that \(z^3 = \overline{z}\).
  • Compute \((1+i)^{132}\).
  • Prove \(\lvert z \rvert\lvert w \rvert = \lvert zw \rvert\).
  • Compute \({n \choose 0} - {n \choose 2} + {n \choose 4} - ...\)

Rotations of Complex Numbers, De Moivre, Roots of Unity

Given \(z = re^{i\theta}\), consider multiplying by \(w = se^{i\phi}\). Using our \(\text{cis }\theta\) definition:

  • \[zw = r\text{cis }\theta * s\text{cis }\phi\]
  • \[= rs(\cos\theta + i\sin\theta)(\cos\phi + i\sin\phi)\]
  • \[= rs(\cos\theta\cos\phi - \sin\theta\sin\phi + i(\sin\theta\cos\phi + \cos\theta\sin\phi))\]
  • \[= rs(\cos(\theta + \phi) + i\sin(\theta + \phi))\]
  • \[= rs\text{ cis }(\theta + \phi)\]

So, multiplying \(z\) by a complex number \(w\) results in a rotation by \(\arg(w)\), and a dilation by \(\lvert w \rvert\).


Here’s an example of the rotation part, in the context of AIME 1994 Problem 8:

Problem: The points \((0, 0) , (a, 11) ,\) and \((b, 37)\) are the vertices of an equilateral triangle. Find the value of \(ab\).

Solution: Consider these points in the complex plane, where our vertices are \(O = 0, X = a + 11i,\) and \(Y = b + 37i\). \(Y\) is a \(60^\circ\) rotation of \(X\) about \(O\), so we can set up:

  • \[(a + 11i)\text{ cis}(60) = b + 37i\]
  • \[\frac{a - 11\sqrt{3}}{2} + \frac{11 + a\sqrt{3}}{2}i = b + 37i\]

Equating real and imaginary parts:

  • \[\frac{11 + a\sqrt{3}}{2} = 37 \rightarrow a = 21\sqrt{3}\]
  • \[b = \frac{a - 11\sqrt{3}}{2} = 5\sqrt{3}\]
  • \[ab = \fbox{315}\]

De Moivre’s formula applies this by stating that \([r \text{ cis }\theta]^n = r^n \text{ cis }(n\theta)\). This follows pretty similarly as the above proof.

The \(n\)th roots of unity are the solutions to \(z^n = 1\). In particular, they will be \(1, \zeta, \zeta^2, ... \zeta^{n-1}\), where \(\zeta = e^{i\frac{2\pi}{n}}\) And since the roots are evenly spaced rotations of each other along a unit circle, they will map to the vertices of a regular \(n\)-gon.

Exercises:

  • A primitive \(n\)th root of unity \(\zeta\) is a complex number that satisfies \(\zeta^n = 1\) and \(\zeta^k \neq 1\) for all positive integers \(k < n\). Find the number of \(2022\)nd primitive roots of unity.
  • (AHSME 1984) If \(\omega = \cos 40^\circ + i\sin 40^\circ\), find \(\lvert \omega + 2\omega^2 + ... + 9\omega^9 \rvert^{-1}\).
  • Let \(1, \zeta, ..., \zeta^{n-1}\) be the \(n\)th roots of unity. Prove that \(1 + \zeta + ... + \zeta^{n-1} = 0\).

Gaussian Integers and Primes

Gaussian integers \(\mathbb{Z}[i]\) form a subring of \(\mathbb{C}[i]\), and all elements are of the form \(\text{a} + \text{b}i\) where \(a, b \in \mathbb{R}\). They are the lattice points of the complex plane.

More Definitions:

  • We say a Gaussian integer \(z\) divides \(w \in \mathbb{Z}[i]\) if there exists \(u \in \mathbb{Z}[i]\) such that \(w = u*z\).
  • A Gaussian integer is invertible if \(\frac{1}{z}\) is also a Gaussian integer, or that it has a multiplicative inverse within \(\mathbb{Z}[i]\).
  • Note that \(\lvert z \rvert \in \mathbb{Z}\) \(\forall z \in \mathbb{Z}[i]\).
  • \(\operatorname{N}(z)\) is defined as \(\lvert z\rvert^2\).

One property is that \(\pm 1\) and \(\pm i\) are the only invertible Gaussian integers, which seems evident by the fact that squares of norms \(\operatorname{N}(\frac{1}{z})\operatorname{N}(z)= \lvert 1\rvert^2 = 1\) cannot both be integral if they are not both equal to \(1\). This implies \(z = a+bi\) has one of \((a,b)\) equal to \(0\), and the other \(\pm 1\), as \(a, b \in \mathbb{Z}\). And we can check that \(\pm 1, \pm i\) all have multiplicative inverses in \(\mathbb{Z}[i]\). We can also see that all four of these will divide any Gaussian integer, just potentially switching signs or coefficients.

  • We say \(z\) is a Gaussian prime if its only Gaussian prime factors are \(z\epsilon\) and \(\epsilon\), where \(\epsilon \in \{ -i, i, -1, 1\}\).

Example 1: Show that \(2\) is not a Gaussian prime.

Solution: To disprove this, we need to find two complex Gaussian factors, neither of which are invertible, that will multiply to \(2\). This will be \((1+i)(1-i)\) or \((-1+i)(-1-i)\).

Example 2: Show that \(3\) is a Gaussian prime.

Solution: If \(3 = wz, \lvert 3 \rvert^2 = \lvert w \rvert^2 \lvert z \rvert^2\). So, to try to disprove this, we will need two complex Gaussian factors, neither of which are invertible, whose squares of norms multiply to \(9\). They must either have norms \(\pm 1\) and \(\pm 9\), or each \(\pm 3\). The first case doesn’t work because a Gaussian integer with norm \(\pm 1\) must be invertible, and the second case does not either because we cannot find an integer sum of squares equal to \(3\).


Something that you may have noticed is that each Gaussian integer has a unique factorization if you do not pay attention to factors of invertible Gaussian integers. For instance, \(2 = (1+i)(1-i)\). There exists a unique Gaussian prime factorization for every \(\mathbb{Z}[i]\):

  • As a first step, we can show that arbitrary Gaussian integer \(z\) will have some finite expansion of Gaussian integer factors. If \(z\) is prime, we are done. Otherwise, we know we can divide \(z = w_1*w_2\) for \(w_1, w_2 \in \mathbb{Z}[i]\), where \(\lvert w_1 \rvert\) and \(\lvert w_2 \rvert\) are strictly less than \(\lvert z \rvert\). Now, repeat for the two new factors. The fact that any future factorization will result in two factors that are at least \(1\) less in norm than your prior value forces your expansion to terminate, as \(\lvert z \rvert\) is only finitely large.
  • Now, we must prove this is unique. So, assume we have two unique expansions \(z = w_1 * w_2 * ... * w_j = v_1 * v_2 * ... * v_k\), where all \(w_i, v_i\) are Gaussian primes. Now consider \(w_1\). It must be true that \(w_1 \lvert v_1 * ... v_k\). But since all values we are working with are Gaussian primes, it must be the case that \(w_1 = \epsilon v_l\) for some \(l \in [1, k]\). Disregarding \(\epsilon\), we can cancel in our equation to get \(w_2 * ... w_j = v_1 * ... * v_{l-1} * v_{l+1} * ... * v_k\). We can repeat this for every \(w_i\), at which point we have to have canceled every factor on the right as well to get an expression equivalent to \(1\). So, the two are equivalent.

Sums of Squares and Integer Primes

We can derive many properties about \(4k+1\) and \(4k+3\) primes based on Gaussian integers.

The first is that any \(4k+3\) prime is necessarily a Gaussian prime, which we can show in a way similar to example 2. If \(p\) is an integer prime \(\equiv 3 \text{ mod } 4\) and \(p = z * w\) for \(z, w \in \mathbb{Z}[i]\), we must have \(\operatorname{N}(z) = \operatorname{N}(w) = p\), otherwise one of them is equal to \(1\). But a sum of integer squares cannot be \(3 \text{ mod } 4\), so we are done.

The second is that any \(4k+1\) prime is not a Gaussian prime, and can be written as a sum of squares. The key part to this proof is that \(-1\) is a quadratic residue \(\text{mod } p\), which can be shown with Wilson’s theorem:

  • \[-1 \equiv (p-1)! = 1 * 2 * ... * \frac{p-1}{2} * \frac{p+1}{2} * ... * (p-1)\]

Note that the latter half of the terms is just the first half with each term negated.

  • \[= (\frac{p-1}{2})! * (-1)^{\frac{p-1}{2}} \equiv (\frac{p-1}{2})!\]

Which is an integer square. From this, we have that, for some \(x\), \(p \lvert (x^2+1) = (x+i)(x-i)\). But since \(p\) cannot divide \((x+i)\) or \((x-i)\), we require that \(p\) is not Gaussian prime. Therefore, we have \(p = z*w\), and for neither to be invertible, \(\operatorname{N}(z) = \operatorname{N}(w) = p\), or that \(p = a^2+b^2\) for \(z = a+bi\).

And lastly, any \(p = a+bi\),where \(a^2+b^2\) is an integer prime, is a Gaussian prime. This follows directly from setting \(\operatorname{N}(p) = \operatorname{N}(z)*\operatorname{N}(w)\), where we see that one of \(\operatorname{N}(z), \operatorname{N}(w)\) must be \(1\).

Problems

  • Let \(A\) be the set of all Gaussian integers \(z\) such that \(z^4-2016i\) is real. Find \(\sum_{z \in A}\operatorname{N}(z)\).
  • Find the number of Gaussian integers \(z\) with magnitude less than \(10000\) such that there exists a different Gaussian integer \(w\) such that \(z = w^4\).
  • Find all integer solutions to \(x^2+4 = y^3\).
  • Prove that an integer with exclusively \(4k+1\) prime factors is the sum of squares.
  • Prove that an integer with no prime factors of the form \(4k+1\) cannot be the hypotenuse of a pythagorean right triangle.
  • Prove that \(1000009 = 235^2+972^2\) is not a prime.

Resources

  • https://www.maths.nottingham.ac.uk/plp/pmzcw/download/fnt_chap5.pdf
  • http://math.uchicago.edu/~may/REU2012/REUPapers/Woodbury.pdf
  • https://web.evanchen.cc/handouts/cmplx/en-cmplx.pdf
  • https://mathweb.ucsd.edu/~alina/oldcourses/2012/104b/zi.pdf

Categories:

Math   Number Theory